{ "cells": [ { "cell_type": "markdown", "metadata": { "id": "jNemS0qoqll3" }, "source": [ "# Exercise 12.1 - Solution\n", "## Weights and activations of a convolutional network\n", "This task consists of two parts. First, set up and evaluate a\n", "convolutional neural network.\n", " 1. Set up and train a convolutional network on the classification of images (data set CIFAR-10). Train your network to at least 70% test accuracy.\n", " - Plot the confusion matrix. What do you observe?\n", " - Plot several falsely classified images along with the predicted class scores. What kinds of misclassification do you observe, why do they occur?\n", " 2. Plot the filter weights in the first layer of your network and see if you can make any sense of it.\n", " 3. Visualize the activations in the first two layers of your network for two input images of choice and describe what you see.\n", " - Does it meet your expectations?\n" ] }, { "cell_type": "code", "execution_count": 2, "metadata": { "colab": { "base_uri": "https://localhost:8080/" }, "id": "OjF2j1SEqll6", "outputId": "8fa23495-ae15-4621-f5d2-15f4eb2b7cfa" }, "outputs": [ { "name": "stdout", "output_type": "stream", "text": [ "keras version 2.4.0\n" ] } ], "source": [ "import tensorflow as tf\n", "from tensorflow import keras\n", "import numpy as np\n", "import matplotlib.pyplot as plt\n", "\n", "layers = keras.layers\n" ] }, { "cell_type": "markdown", "metadata": { "id": "Jfs9BQ2fqll7" }, "source": [ "### Download CIFAR-10 data" ] }, { "cell_type": "code", "execution_count": 2, "metadata": { "id": "DsfigSRCqll7" }, "outputs": [], "source": [ "(x_train, y_train), (x_test, y_test) = keras.datasets.cifar10.load_data()" ] }, { "cell_type": "code", "execution_count": 3, "metadata": { "id": "1R-D0tllqll8" }, "outputs": [], "source": [ "x_train = (x_train - 128.) / 128.\n", "x_test = (x_test - 128.) / 128." ] }, { "cell_type": "code", "execution_count": 4, "metadata": { "id": "laWs8d_mqll8" }, "outputs": [], "source": [ "y_train_one_hot = tf.keras.utils.to_categorical(y_train)\n", "y_test_one_hot = tf.keras.utils.to_categorical(y_test)" ] }, { "cell_type": "markdown", "metadata": { "id": "LZ4InyZeqll8" }, "source": [ "### Design, train, and evaluate a simple CNN" ] }, { "cell_type": "code", "execution_count": 5, "metadata": { "colab": { "base_uri": "https://localhost:8080/" }, "id": "l-5KGW5hqll9", "outputId": "638bf343-8629-4276-e60d-110fd7aabcb2" }, "outputs": [ { "name": "stdout", "output_type": "stream", "text": [ "Model: \"sequential\"\n", "_________________________________________________________________\n", "Layer (type) Output Shape Param # \n", "=================================================================\n", "conv2d (Conv2D) (None, 32, 32, 16) 1216 \n", "_________________________________________________________________\n", "conv2d_1 (Conv2D) (None, 32, 32, 16) 2320 \n", "_________________________________________________________________\n", "conv2d_2 (Conv2D) (None, 16, 16, 32) 4640 \n", "_________________________________________________________________\n", "conv2d_3 (Conv2D) (None, 16, 16, 32) 9248 \n", "_________________________________________________________________\n", "conv2d_4 (Conv2D) (None, 8, 8, 64) 18496 \n", "_________________________________________________________________\n", "conv2d_5 (Conv2D) (None, 8, 8, 64) 36928 \n", "_________________________________________________________________\n", "global_max_pooling2d (Global (None, 64) 0 \n", "_________________________________________________________________\n", "dropout (Dropout) (None, 64) 0 \n", "_________________________________________________________________\n", "dense (Dense) (None, 10) 650 \n", "=================================================================\n", "Total params: 73,498\n", "Trainable params: 73,498\n", "Non-trainable params: 0\n", "_________________________________________________________________\n", "None\n" ] } ], "source": [ "model = keras.models.Sequential([\n", " layers.Convolution2D(16, kernel_size=(5, 5), padding=\"same\", activation='elu', input_shape=(32, 32, 3)),\n", " layers.Convolution2D(16, kernel_size=(3, 3), padding=\"same\", activation='elu'),\n", " layers.Convolution2D(32, kernel_size=(3, 3), padding=\"same\", strides=(2, 2), activation='elu'),\n", " layers.Convolution2D(32, kernel_size=(3, 3), padding=\"same\", activation='elu'),\n", " layers.Convolution2D(64, kernel_size=(3, 3), padding=\"same\", strides=(2, 2), activation='elu'),\n", " layers.Convolution2D(64, kernel_size=(3, 3), padding=\"same\", activation='elu'),\n", " layers.GlobalMaxPooling2D(),\n", " layers.Dropout(0.5),\n", " layers.Dense(10, activation='softmax')\n", " ])\n", "\n", "print(model.summary())" ] }, { "cell_type": "code", "execution_count": 6, "metadata": { "colab": { "base_uri": "https://localhost:8080/" }, "id": "sRLF6anHqlmA", "outputId": "66edb5b1-f4f7-44ca-e653-2d8d56b55719", "scrolled": true }, "outputs": [ { "name": "stdout", "output_type": "stream", "text": [ "Epoch 1/30\n", "1407/1407 [==============================] - 12s 6ms/step - loss: 1.8360 - accuracy: 0.3233 - val_loss: 1.5533 - val_accuracy: 0.4298\n", "Epoch 2/30\n", "1407/1407 [==============================] - 9s 6ms/step - loss: 1.6050 - accuracy: 0.4152 - val_loss: 1.4613 - val_accuracy: 0.4790\n", "Epoch 3/30\n", "1407/1407 [==============================] - 8s 6ms/step - loss: 1.5268 - accuracy: 0.4452 - val_loss: 1.3447 - val_accuracy: 0.5208\n", "Epoch 4/30\n", "1407/1407 [==============================] - 9s 6ms/step - loss: 1.4726 - accuracy: 0.4673 - val_loss: 1.2960 - val_accuracy: 0.5354\n", "Epoch 5/30\n", "1407/1407 [==============================] - 8s 6ms/step - loss: 1.4288 - accuracy: 0.4818 - val_loss: 1.2554 - val_accuracy: 0.5398\n", "Epoch 6/30\n", "1407/1407 [==============================] - 8s 6ms/step - loss: 1.3708 - accuracy: 0.5032 - val_loss: 1.2058 - val_accuracy: 0.5622\n", "Epoch 7/30\n", "1407/1407 [==============================] - 8s 6ms/step - loss: 1.3235 - accuracy: 0.5223 - val_loss: 1.1836 - val_accuracy: 0.5664\n", "Epoch 8/30\n", "1407/1407 [==============================] - 9s 6ms/step - loss: 1.2728 - accuracy: 0.5407 - val_loss: 1.1328 - val_accuracy: 0.6022\n", "Epoch 9/30\n", "1407/1407 [==============================] - 9s 6ms/step - loss: 1.2249 - accuracy: 0.5609 - val_loss: 1.0718 - val_accuracy: 0.6208\n", "Epoch 10/30\n", "1407/1407 [==============================] - 9s 6ms/step - loss: 1.1826 - accuracy: 0.5768 - val_loss: 1.0285 - val_accuracy: 0.6420\n", "Epoch 11/30\n", "1407/1407 [==============================] - 9s 6ms/step - loss: 1.1417 - accuracy: 0.5897 - val_loss: 1.0126 - val_accuracy: 0.6446\n", "Epoch 12/30\n", "1407/1407 [==============================] - 9s 6ms/step - loss: 1.1136 - accuracy: 0.6022 - val_loss: 0.9534 - val_accuracy: 0.6706\n", "Epoch 13/30\n", "1407/1407 [==============================] - 9s 6ms/step - loss: 1.0864 - accuracy: 0.6125 - val_loss: 0.9470 - val_accuracy: 0.6700\n", "Epoch 14/30\n", "1407/1407 [==============================] - 9s 6ms/step - loss: 1.0531 - accuracy: 0.6255 - val_loss: 0.9240 - val_accuracy: 0.6852\n", "Epoch 15/30\n", "1407/1407 [==============================] - 9s 6ms/step - loss: 1.0365 - accuracy: 0.6314 - val_loss: 0.9002 - val_accuracy: 0.6838\n", "Epoch 16/30\n", "1407/1407 [==============================] - 9s 6ms/step - loss: 1.0116 - accuracy: 0.6396 - val_loss: 0.9108 - val_accuracy: 0.6872\n", "Epoch 17/30\n", "1407/1407 [==============================] - 9s 6ms/step - loss: 0.9872 - accuracy: 0.6477 - val_loss: 0.8913 - val_accuracy: 0.6906\n", "Epoch 18/30\n", "1407/1407 [==============================] - 9s 6ms/step - loss: 0.9750 - accuracy: 0.6532 - val_loss: 0.9053 - val_accuracy: 0.6940\n", "Epoch 19/30\n", "1407/1407 [==============================] - 9s 6ms/step - loss: 0.9594 - accuracy: 0.6614 - val_loss: 0.8879 - val_accuracy: 0.6870\n", "Epoch 20/30\n", "1407/1407 [==============================] - 9s 6ms/step - loss: 0.9404 - accuracy: 0.6662 - val_loss: 0.8753 - val_accuracy: 0.6926\n", "Epoch 21/30\n", "1407/1407 [==============================] - 9s 6ms/step - loss: 0.9293 - accuracy: 0.6694 - val_loss: 0.8855 - val_accuracy: 0.6928\n", "Epoch 22/30\n", "1407/1407 [==============================] - 9s 6ms/step - loss: 0.9138 - accuracy: 0.6742 - val_loss: 0.8447 - val_accuracy: 0.7040\n", "Epoch 23/30\n", "1407/1407 [==============================] - 9s 6ms/step - loss: 0.9012 - accuracy: 0.6796 - val_loss: 0.8478 - val_accuracy: 0.7104\n", "Epoch 24/30\n", "1407/1407 [==============================] - 8s 6ms/step - loss: 0.8835 - accuracy: 0.6830 - val_loss: 0.8616 - val_accuracy: 0.7012\n", "Epoch 25/30\n", "1407/1407 [==============================] - 8s 6ms/step - loss: 0.8798 - accuracy: 0.6858 - val_loss: 0.8647 - val_accuracy: 0.6962\n", "\n", "Epoch 00025: ReduceLROnPlateau reducing learning rate to 0.0005000000237487257.\n", "Epoch 26/30\n", "1407/1407 [==============================] - 9s 6ms/step - loss: 0.7754 - accuracy: 0.7240 - val_loss: 0.7802 - val_accuracy: 0.7330\n", "Epoch 27/30\n", "1407/1407 [==============================] - 8s 6ms/step - loss: 0.7374 - accuracy: 0.7356 - val_loss: 0.7827 - val_accuracy: 0.7248\n", "Epoch 28/30\n", "1407/1407 [==============================] - 9s 6ms/step - loss: 0.7168 - accuracy: 0.7403 - val_loss: 0.7784 - val_accuracy: 0.7374\n", "Epoch 29/30\n", "1407/1407 [==============================] - 9s 6ms/step - loss: 0.6996 - accuracy: 0.7496 - val_loss: 0.7995 - val_accuracy: 0.7272\n", "Epoch 30/30\n", "1407/1407 [==============================] - 8s 6ms/step - loss: 0.6914 - accuracy: 0.7500 - val_loss: 0.7921 - val_accuracy: 0.7320\n" ] } ], "source": [ "model.compile(\n", " loss='categorical_crossentropy',\n", " optimizer=keras.optimizers.Adam(1e-3),\n", " metrics=['accuracy'])\n", "\n", "results = model.fit(x_train, y_train_one_hot,\n", " batch_size=32,\n", " epochs=30,\n", " verbose=1,\n", " validation_split=0.1,\n", " callbacks = [keras.callbacks.ReduceLROnPlateau(factor=0.5, patience=3, verbose=1, min_lr=1e-5)],\n", " )" ] }, { "cell_type": "code", "execution_count": 7, "metadata": { "colab": { "base_uri": "https://localhost:8080/", "height": 297 }, "id": "b4kIVpDCqlmA", "outputId": "1ca1597e-facd-4c98-8640-b77c4d86e3c9" }, "outputs": [ { "data": { "image/png": "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\n", "text/plain": [ "
" ] }, "metadata": { "needs_background": "light", "tags": [] }, "output_type": "display_data" } ], "source": [ "plt.figure(1, (12, 4))\n", "plt.subplot(1, 2, 1)\n", "plt.plot(results.history['loss'])\n", "plt.plot(results.history['val_loss'])\n", "plt.ylabel('loss')\n", "plt.xlabel('epoch')\n", "plt.legend(['train', 'val'], loc='upper right')\n", "\n", "plt.subplot(1, 2, 2)\n", "plt.plot(results.history['accuracy'])\n", "plt.plot(results.history['val_accuracy'])\n", "plt.ylabel('accuracy')\n", "plt.xlabel('epoch')\n", "plt.legend(['train', 'val'], loc='upper left')\n", "plt.tight_layout()" ] }, { "cell_type": "markdown", "metadata": { "id": "6Cy3f9RuqlmB" }, "source": [ "### Plot predictions\n", "#### Task\n", "Investigate the predictions of the trained model." ] }, { "cell_type": "code", "execution_count": 8, "metadata": { "id": "EppuDY3LqlmB" }, "outputs": [], "source": [ "def plot_prediction(X, Y, Y_predict, fname=False):\n", " \"\"\"\n", " Plot image X along with predicted probabilities Y_predict.\n", " X: CIFAR image, shape = (32, 32, 3)\n", " Y: CIFAR label, one-hot encoded, shape = (10)\n", " Y_predict: predicted probabilities, shape = (10)\n", " \"\"\"\n", " X = 128 * X + 128\n", " labels = np.array(['airplane', 'automobile', 'bird', 'cat', 'deer',\n", " 'dog', 'frog', 'horse', 'ship', 'truck'])\n", "\n", " fig, (ax1, ax2) = plt.subplots(1, 2, figsize=(8, 4))\n", "\n", " # plot image\n", " ax1.imshow(X.astype('uint8'), origin='upper')\n", " ax1.set(xticks=[], yticks=[])\n", "\n", " # plot probabilities\n", " ax2.barh(np.arange(10), Y_predict, align='center')\n", " ax2.set(xlim=(0, 1), xlabel='Score', yticks=[])\n", "\n", " for i in range(10):\n", " c = 'red' if (i == np.argmax(Y)) else 'black'\n", " ax2.text(0.05, i, labels[i].capitalize(), ha='left', va='center', color=c)\n" ] }, { "cell_type": "code", "execution_count": 9, "metadata": { "colab": { "base_uri": "https://localhost:8080/" }, "id": "HnS-ZNaPqlmC", "outputId": "2dd37188-df07-4722-8765-fe8d5fde44c6" }, "outputs": [ { "name": "stdout", "output_type": "stream", "text": [ "313/313 [==============================] - 1s 3ms/step\n" ] } ], "source": [ "y_pred = model.predict(x_test, verbose=1).squeeze()" ] }, { "cell_type": "code", "execution_count": 10, "metadata": { "colab": { "base_uri": "https://localhost:8080/", "height": 279 }, "id": "-qF-lwqpqlmC", "outputId": "258ebfba-9388-49d5-b98b-76e7b3582694" }, "outputs": [ { "data": { "image/png": "iVBORw0KGgoAAAANSUhEUgAAAdgAAAEGCAYAAADG7YTGAAAABHNCSVQICAgIfAhkiAAAAAlwSFlzAAALEgAACxIB0t1+/AAAADh0RVh0U29mdHdhcmUAbWF0cGxvdGxpYiB2ZXJzaW9uMy4yLjIsIGh0dHA6Ly9tYXRwbG90bGliLm9yZy+WH4yJAAAgAElEQVR4nO3deXxU1f3/8dfJBEjYZRNQIWAFNAshBETjAi5AlcafRay1KsG2Cvyw9ue3VKm1Lm2t/eL21brRr4JUxbUiX2sLLgilYNmMEAGJYFgEZJMlkADJnN8fifmK3HPJjHNnhvB+Ph7zMDlz77mfuTP4ybnzuecYay0iIiISWymJDkBERKQhUoIVEREJgBKsiIhIAJRgRUREAqAEKyIiEoDURAcgIvHXrl07m5GRkegwRJLekiVLtltr20ezrxKsyHEoIyODxYsXJzoMkaRnjFkX7b66RCwiIhIAJVgREZEAKMGKiIgEQAlWREQkAEqwIiIiAVCCFRERCUBEt+k0bd7Stmpz5O1AlshX5DER7wHH98I/cXrxzpPs/Y6FUkPOriJ/j917hMPVEffmEvGZ9NvBEfKWDZ9Ffe+ciDQMESXYVm3aU/TLPxzRHs3//EwU+SIeS+tFdQwb2YWAcDjs15mj1W+f2ElxvZfGO5O0bt3a2VcoJbLzYhzHANi3b59nezTvV6T7VFe7z73rD4zf33x11PfOiUjDoEvEIiIiAVCCFRERCYASrIiISACUYEVERAIQ4WT/FmuPLPiIrjAoil2StcgJR2GQsyv3MVwV2TF97T59uY7jKj+qrKhw9hUKeRcA+Rd5eSsvL4+or3gUP4F/YVZDsGPHDi688EIAtmzZQigUon37muLohQsX0rhx44j7LCoqYtiwYVxxxRUxjVUk2Wg1HRFxatu2LcXFxQDcddddNG/enF/84hd1z1dVVZGaqv+NiHjRvwwRiUhRURFpaWl8+OGHFBQU0LJly8MSb1ZWFm+++SYZGRlMnTqV+++/H2MMOTk5/OUvfzmsrzvuuIMNGzbw9NNPO694iByrlGBFJGIbN25k/vz5hEIh7rrrLs9tPv74Y373u98xf/582rVrx86dOw97fvz48ezdu5fJkyc3+EvtcnxSkZOIRGzEiBFHHXG+9957jBgxgnbt2gHQpk2buud++9vfsnv3bp588kklV2mwlGBFJGLNmjWr+zk1NfWwYrPKysqj7t+vXz+WLFlyxKhWpCFRghWRbyUjI4OlS5cCsHTpUj777DMALrjgAl555RV27NgBcFgyHTp0KLfddhuXXnope/fujX/QInEQ4XewBq8bNqK6xBOHWyLidduF161LNZ25juFzfMdOUdzZ4r7lxicA1/Fd9u/fH9H2ENv3JZa3AkV3K5Qubw4fPpypU6eSmZnJmWeeSY8ePQDIzMzk9ttv5/zzzycUCtGnTx+mTJlSt9+IESPYu3cvhYWFvPXWW6SnpyfoFYgEw0TyP5VOXU61I8ffe0S7M8H4CSfrPa3RHCf4ifjDUZyvSO9pBUiJ8LXEK5FF+odPohPsf/36hiXW2vwoOoyL/Px8u3jx4kSHIZL0jDFR/1vWJWIREZEAKMGKiIgEQAlWREQkAEqwIiIiAYh4sv+oZulvEHxet3EV1ESzooGrmCc5q1UTPUlAoheA0BwJIuKiEayIiEgAlGBFREQCoAQrchxa/vnuRIcg0uApwYqIiARACVZERCQAEa8HazwqY32rLB3tNi7VyLE8hk9fzqdiWWIazfSCjn0SXAjuqjyO19SWkVKhsIhEQyNYETmq3//+92RmZpKTk0Nubi7//ve/ycjIYPv27UdsO2PGDO67774ERCmSXCIewYrI8WXBggW8+eabLF26lCZNmrB9+3YOHjzo3L6wsJDCwsI4RiiSnDSCFRFfmzdvpl27djRp0gSAdu3a0blzZwAeffRR8vLyyM7OZtWqVQBMmTKFcePGAVBUVMTo0aPJz8+nR48evPnmm4l5ESIJoAQrIr4GDx7Mhg0b6NGjB2PHjmXOnDl1z7Vr146lS5cyZswY7r//fs/9y8rKWLhwIX/7298YPXo0lZWV8QpdJKGUYEXEV/PmzVmyZAmTJk2iffv2/OAHP6hbOP373/8+AH379qWsrMxz/yuvvJKUlBROO+00unfvXjfSFWno9B2siBxVKBRi4MCBDBw4kOzsbJ599lmAusvGoVCIqqoqz32/WTWe6PmrReIl8hGstUc8DO4HDebhx8ThEcOY/Q6TQMYY5yORov1UNBSffPIJpaWldb8XFxfTtWvXeu//yiuvEA6HWbNmDWvXrqVnz55BhCmSdDSCFRFf5eXl3HTTTezatYvU1FS+853vMGnSpHoXLHXp0oX+/fuzZ88ennzySdLS0gKOWCQ5KMGKiK++ffsyf/78I9q//p1rfn4+77//PlBTOVxUVFT33EUXXcSTTz4ZcJQiyUdFTiIiIgHQCFZEAvNVtbHI8UgjWBERkQBoBCtyFImuYhaRY5NGsCLHoeyTWiU6BJEGTwlWREQkAEqwIiIiAVCCFRERCYASrIiISABURSzfSjQVttbGbhZf1/H94gqHwxHt4xevKoxFxEUjWBERkQAowYqIr+bNmx/2+5QpUxg3blyCohE5dijBikigXOvEijR0SrAiErWysjIuuOACcnJyuPDCC1m/fj1Qs6LO6NGjOfPMM/nlL3/JnDlzyM3NJTc3lz59+rB3714AJk6cSL9+/cjJyeHOO+9M5EsRiTkVOYmIr4qKCnJzc+t+37lzJ4WFhQDcdNNNjBw5kpEjR/LMM8/ws5/9jOnTpwOwceNG5s+fTygU4nvf+x6PPfYYBQUFlJeXk5aWxqxZsygtLWXhwoVYayksLGTu3Lmcd955CXmdIrGmEayI+EpPT6e4uLjucc8999Q9t2DBAq6++moArr32WubNm1f33IgRIwiFQgAUFBRwyy238Mgjj9Qt3D5r1ixmzZpFnz59yMvLY9WqVZSWlsb3xYkEKIoRrNdtCbG77UJcXLeD6NwHzvdWHN2m49KsWbO6n2+77TYuvfRS3nrrLQoKCpg5cybWWiZMmMCNN96YwChFgqMRrIhE7eyzz+bFF18E4Pnnn+fcc8/13G7NmjVkZ2dz66230q9fP1atWsWQIUN45plnKC8vB+Dzzz9n69atcYtdJGj6DlZEovboo48yatQoJk6cSPv27Zk8ebLndg8//DCzZ88mJSWFzMxMvvvd79KkSRNWrlzJWWedBdTcDvTcc8/RoUOHeL4EkcCYSGbV6dSluy36xe+PaLfWe2YcP7GczSeRx4jXccJh1zFie+wU53FiJ5rzFctz7JrJyXlsn+eM4xLxw7f/dIm1Nj+iA8VRfn6+Xbx4caLDEEl6xpio/y3rErGIiEgAlGBFREQCEPl3sMbjglkUl+9chZn+PTmedTRH89eDdR7DZ8J3R3vYFYFPXynG+/JldbX3bDgmxf0qU0zkx49PVXI0x4hsH7/aXtcZi+q9VxGxiDhoBCsiIhIAJVgREZEAKMGKiIgEQAlWREQkAEqwIiIiAYhiJiePKlevyuIo+RVlOicbcOyUEkVYrmMYnypWV4XvunWbvY8Rrnb21b3bSZ7tVQcPerY3btzE2VdKyPvEOKtlj/Jc7MSuitj1efH/SEZWLWy8PvN1Ip9kRUSODxrBioiIBEAJVkREJABKsCIiIgHQajoi4isUCpGdnV33+/Tp08nIyEhcQCLHCCVYEfGVnp5OcXGx53PWWqy1pPhM2SlyvNK/ChGJSFlZGT179uS6664jKyuLDRs2MH78eLKyssjOzuall14CapYFHDt2LL169eLiiy/mkksu4dVXX01w9CLx0yBGsHGZb91nVvcmjRt5tu/etsWzfeeXO519tWzR2LM9Lb25Z3u81ryNRjLHJvVXUVFBbm4uAN26deOhhx6itLSUZ599lgEDBvDaa69RXFzMRx99xPbt2+nXrx/nnXce//rXvygrK2PFihVs3bqV008/neuvvz7Br0YkfhpEghWR4HzzEnFZWRldu3ZlwIABAMybN48f/vCHhEIhTjzxRM4//3wWLVrEvHnzGDFiBCkpKXTs2JFBgwYl6iWIJIQuEYtIxJo1a5boEESSnhKsiHwr5557Li+99BLV1dVs27aNuXPn0r9/fwoKCnjttdcIh8N88cUXvP/++4kOVSSudIlYRL6Vyy+/nAULFtC7d2+MMfznf/4nHTt2ZPjw4bz77rucccYZnHLKKeTl5dGqVatEhysSN0qwIuKrvLz8sN8zMjIoKSmp+90Yw8SJE5k4ceJh26WkpHD//ffTvHlzduzYQf/+/Q+7n1akoTumEqxxVfI6i1VjWcXqriLes+tLz/ZD5Xs821PD3osDAGzdstWzvd2JIc/29PSmzr4Szfl+xbIv11ts3ZPwq7o5foYNG8auXbs4ePAgd9xxBx07dkx0SCJxc0wlWBE5tuh7VzmeqchJREQkAEqwIiIiAVCCFRERCYASrIiISACiKHI6sprT+lRsOntxVIX6rsrh2Mc4ikJTXE/4cFWYhkLeVbwAe3Z7VwtX7Paec9j6VCR/udO7Irlpy9ae7e1jvIqJuyDb+5mqKndFdHV1dUTtfn05wwp7x5XW2HtOZ4AmTZp496XqYhGJIY1gRUREAqAEKyIiEgAlWBERkQAowYqIiARACVZEfIVCIXJzc8nMzKR379488MADhMORFzaKHG80VaKI+Pr6gutbt27l6quvZs+ePdx9990JjkwkuUWcYI3HjRxVBw86t688UOnZ7rq9ovLAAWdfrts4XDe9pPoM0FNSvPcKO24h8bt9qNrx+lu2aO7ZXr5vn7OvfXt2e7Zv/Xy9Z/uBfXvdcTnuOqmq8n6NAFXV3ue46tAh72P4jGRct7242tu2bevsq0OHDp7tqY28P8KhkPv9ivS2MuuzaETsljM4NnTo0IFJkybRr18/7rrrLg4cOMCYMWNYvHgxqampPPjggwwaNIj9+/dTVFRESUkJPXv2ZNOmTTz22GPk5+cn+iWIxI1GsCISke7du1NdXc3WrVt57rnnMMawfPlyVq1axeDBg1m9ejWPP/44J5xwAitWrKCkpITc3NxEhy0Sd/oOVkSiNm/ePK655hoAevXqRdeuXVm9ejXz5s3jqquuAiArK4ucnJxEhimSEEqwIhKRtWvXEgqFnJftRaSGEqyI1Nu2bdsYPXo048aNwxjDueeey/PPPw/A6tWrWb9+PT179qSgoICXX34ZgBUrVrB8+fJEhi2SEPoOVkR8VVRUkJuby6FDh0hNTeXaa6/llltuAWDs2LGMGTOG7OxsUlNTmTJlCk2aNGHs2LGMHDmSM844g169epGZmUmrVq0S/EpE4ivCBGsx4SOrTFPd8+CT5qjydN1Ht2e3u8J227Zt3lFVe/d1YN9+Z19ffuk9qf4BRxVzyKeKOBXvqty0xo082w9Wuauuwfu5UKX3Sf5spXd1McCX+7xfiwm5J8Lv2OlEz/bmzb0rotu2a+Psq2mzpp7tIcdnonkz72MANEnznqDfWeDrU90cDruqqKOZ7L/h1xG7FmcASEtLY/LkyZ7tzz33HGlpaaxZs4aLLrqIrl27BhmmSNLRCFZEYm7//v0MGjSIQ4cOYa3l8ccfp7HPCkciDZESrIjEXIsWLVi8eHGiwxBJKBU5iYiIBEAJVkREJABKsCIiIgGIuIrYq9LSr8I21MS7sMEY7+rLU04+2dlXp44d/cP7hkMV7nmNPy4p8WzfvGVzxMeuPuh9HNc8udsd1dAAByq8K58POOZhPumUU5x9ta32fl/2VbrPS59c7xl3XFXEoZC7hNw45nuudsxF7LdCi2v+YiJtr3nS57kIjh1xTyJyPNEIVkREJABKsCIiIgFQghUREQmAEqyIiEgAlGBFREQCoAQrIr5CoRC5ublkZmbSu3dvHnjgAd+KbxGpEfhUia5bHFztrtt3ABo18p4839VX45be2wNk9c72bO/avZt3X46J+wEq9nkvUNCuXTvP9j179jj7+mztWs/2RiHv89L+RPftQ4eqvc9Lm3btnfu4FihwvS2+t9aEHe+9ozO/22HcB/FujtcU/A1/qn9IT0+nuLgYgK1bt3L11VezZ88e7r777m/Vr7UWay0pPrf5iRzL9MkWkXrr0KEDkyZN4k9/+hPWWqqrqxk/fjz9+vUjJyeHp556qm7biRMn1rXfeeedAJSVldGzZ0+uu+46srKy2LBhQ6JeikjgNNm/iESke/fuVFdXs3XrVt544w1atWrFokWLOHDgAAUFBQwePJjS0lJKS0tZuHAh1loKCwuZO3cuXbp0obS0lGeffZYBAwYk+qWIBEoJVkSiNmvWLJYtW8arr74KwO7duyktLWXWrFnMmjWLPn36AFBeXk5paSldunSha9euSq5yXFCCFZGIrF27llAoRIcOHbDW8uijjzJkyJDDtpk5cyYTJkzgxhtvPKy9rKyMZs2axTNckYTRd7AiUm/btm1j9OjRjBs3DmMMQ4YM4YknnuDQoUMArF69mn379jFkyBCeeeYZysvLAfj888/ZunVrIkMXibvAR7CuquBIq4ujUeVT4ZrWtKlne1PHpPZbtmxx9vXl3r2e7e07dfJsP6F9B2dfzVu39mx3VVpu2+peOKCywjuu9KZpzn1sVbV3exRvi3HU2EbzDjurzqPoK9LP5PFQKeynoqKC3NxcDh06RGpqKtdeey233HILAD/5yU8oKysjLy8Pay3t27dn+vTpDB48mJUrV3LWWWcBNYtFPPfcc76LQ4g0NLpELCK+qqu9/+iCmj/87r33Xu69994jnrv55pu5+eabj2gvcaxkJdLQ6BKxiIhIAJRgRUREAqAEKyIiEgAlWBERkQAowYqIiASgQVcR+91eEXbckhF2VEye0KaNs69mLVp6tlvj/feL3+1DJuT9loQdr6ZNB/ctPye09Y45XO1zfOczwfM7duxu3oq8N7+tj/dbeETETSNYERGRACjBioiIBEAJVkREJABKsCIiIgFQghUREQlATKqIrXVXpboYR2lmNFWZzipPnxnqIz1OSsj9t0goxfs02vChKI7tmiDfu901oX5NYI6J1WO4oEI0izOkOHbx6yklwuP4bx5ZX6oUrlns4uc//zmLFi2idevWnHjiiTz88MP06NHjiG137drFCy+8wNixYxMQqUjy0AhWRHxZa7n88ssZOHAga9asYcmSJfzhD3/giy++8Nx+165dPP7443GOUiT5KMGKiK/Zs2fTqFEjRo8eXdfWu3dv+vTpw4UXXkheXh7Z2dm88cYbANx2222sWbOG3Nxcxo8fn6iwRRKuQU80ISLfXklJCX379j2iPS0tjddff52WLVuyfft2BgwYQGFhIffddx8lJSUUFxcnIFqR5KEEKyJRsdbyq1/9irlz55KSksLnn3/uvGwscjxSghURX5mZmbz66qtHtD///PNs27aNJUuW0KhRIzIyMqisrExAhCLJKYrvYK3HI3LG8YhlXzF9WOt8EK72fBhsFI+w5yPFWs+HsWGfhyPmGJ//WPE9xxHz+pzWPKz1fvgE5n44j9NwXHDBBRw4cIBJkybVtS1btox169bRoUMHGjVqxOzZs1m3bh0ALVq0YO/evYkKVyRpqMhJRHwZY3j99dd55513OPXUU8nMzGTChAlccsklLF68mOzsbKZOnUqvXr0AaNu2LQUFBWRlZanISY5rukQsIkfVuXNnXn755SPaFyxY4Ln9Cy+8EHRIIklPI1gREZEAKMGKiIgEQAlWREQkAEqwIiIiAUhYkZPrRoZE3yoSzeT1sQ0gDoeI4jXG8rwk+hwbE9mnzPcOnkR/YEUkaWkEKyIiEgAlWBERkQAowYqIiARACVZERCQASrAiIiIBiLiKOGb1n47SzERPk+6ucE1wFW2cKm/jUS2c6CrmWB4j0RXRIpK8NIIVEV+hUIjc3Fx69+5NXl4e8+fPB2DTpk1cccUV9epj4MCBLF68OMgwRZKOJvsXEV/p6ekUFxcDMHPmTCZMmMCcOXPo3Lmz5zqxVVVVpKbqfy0i+lcgIvW2Z88eTjjhBADKysoYNmwYJSUlTJkyhb/+9a+Ul5dTXV3NP/7xD0aNGsVHH31Er169qKioSHDkIvGnBCsivioqKsjNzaWyspLNmzfz3nvveW63dOlSli1bRps2bXjwwQdp2rQpK1euZNmyZeTl5cU5apHE03ewIuLrq0vEq1at4h//+AfXXXedZ3HXxRdfTJs2bQCYO3cu11xzDQA5OTnk5OTENWaRZBDZCNYSt2pWOR7ps5XszjrrLLZv3862bduOeK5Zs2YJiEgkeWkEKyL1tmrVKqqrq2nbtq3vdueddx4vvPACACUlJSxbtiwe4YkkFX0HKyK+vvoOFmru+3322WcJhUK++4wZM4ZRo0Zx+umnc/rpp9O3b994hCqSVJRgRcRXdXW1Z3tGRgYlJSUAFBUVUVRUVPdceno6L774YjzCE0laukQsIiISACVYERGRACjBioiIBCDy72Bt2KPNfXuFcbY7nvG5U8M5ebzz4O7OrGMvV7sf52tx9RXNXP94fw/md+6jmojeRPa++J4v57oJ0Zxj1/sV+efIKYqFHozrfInIcU8jWBERkQAowYqIiARACVZERCQASrAiIiIBUIIVEREJQIRVxNZRzRlNyaaj+tKnKDPSqtiwdVTexlpM56h3let6VG8fraco9nEVXsd2Gv7Y9RZxdbGvyKuItUCBiLhoBCsiIhIAJVgREZEAKMGKyFFNnz4dYwyrVq066rYPP/ww+/fvDzSeKVOmMG7cOM/nzj77bADKysrIysoKNA4RP0qwInJU06ZN45xzzmHatGlH3TYeCdbP/PnzE3Zska9TghURX+Xl5cybN4+nn366bgm6999/n2HDhtVtM27cOKZMmcIjjzzCpk2bGDRoEIMGDQJqknN2djZZWVnceuutdfs0b96c8ePHk5mZyUUXXcTChQsZOHAg3bt3Z8aMGQBUVlYyatQosrOz6dOnD7Nnz67bf8OGDQwcOJDTTjuNu++++7B+v6m6uprx48fTr18/cnJyeOqpp2J7kkQ8JDDBhj0f1rofrn2cD2udD+t4RMMdc7Xnwz9u63jER9haz8exJ4rPS8TvybF4XiL3xhtvMHToUHr06EHbtm1ZsmSJc9uf/exndO7cmdmzZzN79mw2bdrErbfeynvvvUdxcTGLFi1i+vTpAOzbt48LLriAjz/+mBYtWvDrX/+at99+m9dff53f/OY3ADz22GMYY1i+fDnTpk1j5MiRVFZWArBw4UJee+01li1bxiuvvMLixYudcT399NO0atWKRYsWsWjRIv785z/z2WefxfAsiRxJI1gR8TVt2jSuuuoqAK666qp6XSb+yqJFixg4cCDt27cnNTWVH/3oR8ydOxeAxo0bM3ToUACys7M5//zzadSoEdnZ2ZSVlQEwb948rrnmGgB69epF165dWb16NQAXX3wxbdu2JT09ne9///vMmzfPGcesWbOYOnUqubm5nHnmmezYsYPS0tKIz4VIJCJfTUdEjhs7d+7kvffeY/ny5RhjqK6uxhjDZZddRjj8v/dZfzWqjESjRo3qViNKSUmhSZMmdT9XVVUddf9vrmTkt7KRtZZHH32UIUOGRBynSLQ0ghURp1dffZVrr72WdevWUVZWxoYNG+jWrRvhcJgVK1Zw4MABdu3axbvvvlu3T4sWLdi7dy8A/fv3Z86cOWzfvp3q6mqmTZvG+eefX+/jn3vuuTz//PMArF69mvXr19OzZ08A3n77bXbu3ElFRQXTp0+noKDA2c+QIUN44oknOHToUF1f+/bti/h8iERCI1gRcZo2bdphhUkAw4cP58UXX+TKK68kKyuLbt260adPn7rnb7jhBoYOHVr3Xex9993HoEGDsNZy6aWXctlll9X7+GPHjmXMmDFkZ2eTmprKlClT6ka6/fv3Z/jw4WzcuJFrrrmG/Px8Zz8/+clPKCsrIy8vD2st7du3r/suWCQoJpLCnk6ndLXX33L7Ee3RFAc5p+TzXXA9sqn/wj7buw4T1QLaYddxYlcEE805Djvjivw4sVxYPNpisqCPEdXn2HFeHrrjP5ZYa93/x0+w/Px861cUJCI1jDFR/1vWJWIREZEARHyJOBw+8q/86CaVj3xi9YhHGMb990Pk4zGfY5s4LCpgYzeC9BPLkWosud97R7vrEonfPs5j+zwXTs7zJSKJpxGsiIhIAJRgRUREAqAEKyIiEgAlWBERkQAowYqIiAQgiokmYnQPo4m88vjYm1w9lvEmZ7VqPO5p9eU8LdHEFc0+yfm+HM3yz3eTcdvfEh2GSIOmEayIiEgAlGBFREQCoAQrIiISACVYETmqwasXUPbHYZy6YwMAHfbu4PHX7424n3lPXM8J+3fHOjyRpKQEKyJHVbhyLgtPPoPCFTWLpW9t0Zaxl//qiO1C4ThMGypyjIiwitgCHv+AfCtJI59z2N1VpPv4bR9p9afvMj9RHD9S0VRdR7OPg/MlRrECTRSHd89d7S2q6mbXPn5vfVTV8MeWpgcryN/4MT/84R/479fu4aFzf8TJu7/g6VfvZsiPH+eK5e8wZPV8mh2sJGTDPHTO1fy/fz7PvsbpdN21mQ+6ZPPrwWOx35gbfNJff0enPdtoUnWIyfmFTMsdCsDHD17B5PxCLvx0IZWpTfjp8F+zvdkJtNm/m9/PfIzOe7YBcM+FP2XJyWfE/XyI1JdGsCLi6+LSD5jTrS+ftTmJXWktyNry6RHbZG1Zw5j/M4EfXH0fALmbV3PnxaO5+MeP0+XLLQz9ZP4R+4z/7s18r+i/+N7IhyhaMoPWFXsAaHaokg879+S71/+Jf5+SyVUfzQTgzncm8XS/y7hs5EOMuXwCf/z7IwG+apFvTwuui4ivwpVzmdy3EID/Of08ClfMYWrfYYdtMy+jD7vTW9T9XtypBxtadwRgxhnn0W/jCv7e65zD9hm1ZAZDVi8AoNOe7XTbuYkPT2rJgVAq757aH4CSjt/hnLJiAArWFXPajvV1+zc/uJ+mByvY3zg9xq9YJDaUYEXEqVXFXs5et+9PnZQAAAfzSURBVIye28oAQ4oNYzFMzbv0sO32N25y+I7fWPbQfuP3AeuXUVD2EZdfez+VjdJ48YXbaFJ9EICqlNS6/atNCqm13+umWMvl1z7AgdTGsXuBIgHSJWIRcbrkk3/xeuYgzhkzmXPGPMPZY6ewsfWJdN673Xe/3ptXc/KuLRgb5nsr/8mib3xX2uLAfnanNaOyURqn7thAn02fHDWWf2b0YeSS/6n7/Ywv1kb3okTiRAlWRJwKV85hZo+zDmv7e4+zGbvgFd/9Pup4Gve8/STv/PcYNrQ+8Yg+5nTrS2o4zDt/Hs2tc57lw849jxrLXRfdQM6WT/n7M+N4+7/H8KPityJ/QSJxZCKptux0Shdb9PNbj2i34ciriE0UFbaRVoZa33rVWFYRuypJY1dFbG3ktbfhcPBVxNFU60Y1e6/rOI7OrPM98TuE9zH8XuI3L31+5b/uvHWJtTY/4iDipEmn02ynkQ8H0veA9cv46cLX+fEVdwbSv0g8rfvjsKj/LUf8HWyKOfL/NuEoblXw6Cbm/JN4DJOfTwSx4vj/+FF2inwXZ8J09OUXl+sp4/sHWYSdOfj9cRVpIvX/dB9rC1CISLyoyElEYuqDLjl80CUn0WGIJJy+gxUREQmARrAix6Hsk1qx+L5Lj76hyHHO/DH6fTWCFRERCYASrIiISAAiukS8ZeOG7ff+x03rggpGpAHpmugARCSxIkqw1tr2QQUiIiLSkOgSsYiISACUYEVERAKgBCsiIhIAJVgREZEAKMGKiIgEIKLVdESkYTDG7AWOvghrYrUD/BeeTaxkjw8UYyz0tNa2iGZHTZUocnz6JJmX0wMwxixO5hiTPT5QjLFgjFkc7b66RCwiIhIAJVgREZEAKMGKHJ8mJTqAekj2GJM9PlCMsRB1fCpyEhERCYBGsCIiIgFQghUREQmAEqxIA2aMGWqM+cQY86kx5jaP55sYY16qff7fxpiMJIvvFmPMCmPMMmPMu8aYuC8DeLQYv7bdcGOMNcbE/ZaT+sRojLmy9lx+bIx5IZniM8Z0McbMNsZ8WPteXxLn+J4xxmw1xpQ4njfGmEdq419mjMmrV8fWWj300KMBPoAQsAboDjQGPgLO+MY2Y4Ena3++CngpyeIbBDSt/XlMPOOrb4y127UA5gIfAPnJFiNwGvAhcELt7x2SLL5JwJjan88AyuJ8Ds8D8oASx/OXAH8HDDAA+Hd9+tUIVqTh6g98aq1da609CLwIXPaNbS4Dnq39+VXgQmOMSZb4rLWzrbX7a3/9ADg5TrHVO8ZavwX+CFTGM7ha9Ynxp8Bj1tovAay1W5MsPgu0rP25FbApjvFhrZ0L7PTZ5DJgqq3xAdDaGNPpaP0qwYo0XCcBG772+8baNs9trLVVwG6gbVyiq198X/djakYR8XTUGGsvF55irf1bPAP7mvqcxx5AD2PMv4wxHxhjhsYtuvrFdxdwjTFmI/AWcFN8Qqu3SD+rgKZKFJFjgDHmGiAfOD/RsXydMSYFeBAoSnAoR5NKzWXigdRcBZhrjMm21u5KaFT/64fAFGvtA8aYs4C/GGOyrLXhRAf2bWgEK9JwfQ6c8rXfT65t89zGGJNKzeW5HXGJrn7xYYy5CLgdKLTWHohTbF85WowtgCzgfWNMGTXfz82Ic6FTfc7jRmCGtfaQtfYzYDU1CTdZ4vsx8DKAtXYBkEbNIgDJol6f1W9SghVpuBYBpxljuhljGlNTxDTjG9vMAEbW/nwF8J6trepIhviMMX2Ap6hJrvH83rBeMVprd1tr21lrM6y1GdR8T1xorY16gvhYx1hrOjWjV4wx7ai5ZLw2ieJbD1xYG9/p1CTYbXGKrz5mANfVVhMPAHZbazcfbSddIhZpoKy1VcaYccBMaio5n7HWfmyMuQdYbK2dATxNzeW4T6kp8rgqyeKbCDQHXqmtvVpvrS1MshgTqp4xzgQGG2NWANXAeGttXK5U1DO+/wD+bIz5f9QUPBXF8Q89jDHTqPkDpF3t98B3Ao1q43+Smu+FLwE+BfYDo+rVbxxfg4iIyHFDl4hFREQCoAQrIiISACVYERGRACjBioiIBEAJVkREJABKsCIiATDG3F67cs0yY0yxMebMRMck8aX7YEVEYqx2ur9hQJ619kDt5A6Nv0V/qbVzRcsxRCNYEZHY6wRs/2pqR2vtdmvtJmNMP2PMfGPMR8aYhcaYFsaYNGPMZGPM8tr1UAcBGGOKjDEzjDHvAe8aY5rVrlu6sHY7r1V9JIloBCsiEnuzgN8YY1YD7wAvAQtq//sDa+0iY0xLoAK4GbDW2mxjTC9gljGmR20/eUCOtXanMeZeaqayvN4Y0xpYaIx5x1q7L94vTupHI1gRkRiz1pYDfYEbqJlT9yXgRmCztXZR7TZ7ai/7ngM8V9u2ClhHzVzBAG9ba79ap3QwcJsxphh4n5r5ervE5QVJVDSCFREJgLW2mppE+L4xZjnwf6Po5uujUwMMt9Z+EoPwJA40ghURiTFjTE9jzNeXg8sFVgKdjDH9ardpUbtE4D+BH9W29aBmVOqVRGcCN5naVQ9qVxqSJKYRrIhI7DUHHq39rrSKmlVYbgAm17anU/P960XA48ATtaPcKmpWkjlQm0e/7rfAw8Cy2oXeP6OmUlmSlFbTERERCYAuEYuIiARACVZERCQASrAiIiIBUIIVEREJgBKsiIhIAJRgRUREAqAEKyIiEoD/DzaMIpXpg3kDAAAAAElFTkSuQmCC\n", "text/plain": [ "
" ] }, "metadata": { "needs_background": "light", "tags": [] }, "output_type": "display_data" } ], "source": [ "idx_1 = np.random.choice(len(x_test), 1)[0]\n", "plot_prediction(x_test[idx_1], y_test_one_hot[idx_1], y_pred[idx_1])" ] }, { "cell_type": "code", "execution_count": 11, "metadata": { "colab": { "base_uri": "https://localhost:8080/", "height": 279 }, "id": "oamxb5utqlmD", "outputId": "30141833-4617-4fc9-8298-14481f91163e" }, "outputs": [ { "data": { "image/png": "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\n", "text/plain": [ "
" ] }, "metadata": { "needs_background": "light", "tags": [] }, "output_type": "display_data" } ], "source": [ "idx_2 = np.random.choice(len(x_test), 1)[0]\n", "plot_prediction(x_test[idx_2], y_test_one_hot[idx_2], y_pred[idx_2])" ] }, { "cell_type": "markdown", "metadata": { "id": "H5YkKktKqlmD" }, "source": [ "### Plot confusion matrix\n", "#### Task\n", "Plot the confusion matrix and comment on your findings" ] }, { "cell_type": "code", "execution_count": 12, "metadata": { "id": "yNT0dPgFqlmD" }, "outputs": [], "source": [ "def plot_confusion(Y_true, Y_predict, fname=False):\n", " \"\"\"\n", " Plot confusion matrix\n", " Y_true: array of true classifications (0-9), shape = (N)\n", " Y_predict: array of predicted classifications (0-9), shape = (N)\n", " \"\"\"\n", " labels = np.array(['airplane', 'automobile', 'bird', 'cat', 'deer',\n", " 'dog', 'frog', 'horse', 'ship', 'truck'])\n", " C = np.histogram2d(Y_true, Y_predict, bins=np.linspace(-0.5, 9.5, 11))[0]\n", " Cn = C / np.sum(C, axis=1)\n", "\n", " fig = plt.figure(figsize=(8, 8))\n", " plt.imshow(Cn, interpolation='nearest', vmin=0, vmax=1, cmap=plt.cm.YlGnBu)\n", " plt.colorbar()\n", " plt.xlabel('prediction')\n", " plt.ylabel('truth')\n", " plt.xticks(range(10), labels, rotation='vertical')\n", " plt.yticks(range(10), labels)\n", "\n", " for x in range(10):\n", "\n", " for y in range(10):\n", " plt.annotate('%i' % C[x, y], xy=(y, x), ha='center', va='center')\n", "\n", " plt.tight_layout()" ] }, { "cell_type": "code", "execution_count": 13, "metadata": { "colab": { "base_uri": "https://localhost:8080/", "height": 559 }, "id": "H7MNkwvDqlmE", "outputId": "55e6d0db-faff-457f-dc0a-19d69f4b73e6" }, "outputs": [ { "data": { "image/png": "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\n", "text/plain": [ "
" ] }, "metadata": { "needs_background": "light", "tags": [] }, "output_type": "display_data" } ], "source": [ "y_predict_cl = np.argmax(y_pred, axis=1)\n", "y_test_cl = np.argmax(y_test_one_hot, axis=1)\n", "\n", "plot_confusion(y_test_cl, y_predict_cl)" ] }, { "cell_type": "markdown", "metadata": { "id": "yNqTA9DgqlmE" }, "source": [ "*The most misclassifications occur for classes that share similar features in the CNN.\n", "Samples of these classes have similar shapes, colors, and structures.\n", "Thus, most misclassifications occur e.g., for cars and trucks, for birds and airplanes (wings), for cats and dogs, or for airplanes and ships (similar background).*" ] }, { "cell_type": "markdown", "metadata": { "id": "4GgHFIlSqlmF" }, "source": [ "### Plot filter weights of the first layer" ] }, { "cell_type": "code", "execution_count": 14, "metadata": { "colab": { "base_uri": "https://localhost:8080/", "height": 544 }, "id": "sOGzYkj2qlmF", "outputId": "9dc8c77f-2ddd-4d29-cb2e-97f9e4301001" }, "outputs": [ { "data": { "text/plain": [ "Text(0.5, 0.98, '36 convolutional filters')" ] }, "execution_count": 14, "metadata": { "tags": [] }, "output_type": "execute_result" }, { "data": { "image/png": "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\n", "text/plain": [ "
" ] }, "metadata": { "needs_background": "light", "tags": [] }, "output_type": "display_data" } ], "source": [ "# ----------------------------------------------------------\n", "# Plot the filters in the first convolution layer\n", "# ----------------------------------------------------------\n", "conv1 = model.layers[0]\n", "\n", "W1, b1 = conv1.get_weights()\n", "W1 = (W1 * 128 + 128).astype(np.int)\n", "\n", "nx, ny, nc, nf = W1.shape\n", "n = np.ceil(nf**.5).astype(int)\n", "fig, axes = plt.subplots(n, n, figsize=(7, 7))\n", "fig.subplots_adjust(left=0.05, bottom=0.05, right=0.95, top=0.95, hspace=0, wspace=0)\n", "\n", "\n", "for i in range(n**2):\n", " ax = axes.flat[i]\n", "\n", " if i < nf:\n", " ax.imshow(W1[..., i], origin='upper')\n", " ax.xaxis.set_visible(False)\n", " ax.yaxis.set_visible(False)\n", " else:\n", " ax.axis('Off')\n", "\n", "fig.tight_layout()\n", "fig.suptitle('36 convolutional filters', va='bottom')" ] }, { "cell_type": "markdown", "metadata": { "id": "u4f42N94qlmF" }, "source": [ "### Plot activations in convolutional layers\n", "#### Task\n", "Visualize the filters in the first convolutional layer of your CNN." ] }, { "cell_type": "code", "execution_count": 15, "metadata": { "id": "5asAr6KNqlmG" }, "outputs": [], "source": [ "def visualize_activation(A, name='conv'):\n", " nx, ny, nf = A.shape\n", " n = np.ceil(nf**.5).astype(int)\n", " fig, axes = plt.subplots(n, n, figsize=(8, 8))\n", " fig.subplots_adjust(left=0.05, bottom=0.05, right=0.95, top=0.95, hspace=0, wspace=0)\n", "\n", " for i in range(n**2):\n", " ax = axes.flat[i]\n", "\n", " if i < nf:\n", " ax.imshow(A[..., i], origin='upper', cmap=plt.cm.Greys)\n", " ax.xaxis.set_visible(False)\n", " ax.yaxis.set_visible(False)\n", " else:\n", " ax.axis('Off')\n", "\n", " plt.tight_layout()\n", " fig.suptitle(name, va='bottom')\n", " plt.show()" ] }, { "cell_type": "markdown", "metadata": { "id": "EW8RJMngqlmG" }, "source": [ "#### Task\n", "Visualize the activations in the first two layers of your network for two input images of choice and describe what you see." ] }, { "cell_type": "code", "execution_count": 16, "metadata": { "colab": { "base_uri": "https://localhost:8080/", "height": 1000 }, "id": "uwoWZ4GTqlmG", "outputId": "78fa070c-c482-4ef0-c4b9-8bd32e10b8d1", "scrolled": false }, "outputs": [ { "data": { "image/png": "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\n", "text/plain": [ "
" ] }, "metadata": { "needs_background": "light", "tags": [] }, "output_type": "display_data" }, { "data": { "image/png": "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\n", "text/plain": [ "
" ] }, "metadata": { "needs_background": "light", "tags": [] }, "output_type": "display_data" }, { "data": { "image/png": "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\n", "text/plain": [ "
" ] }, "metadata": { "needs_background": "light", "tags": [] }, "output_type": "display_data" }, { "data": { "image/png": "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\n", "text/plain": [ "
" ] }, "metadata": { "needs_background": "light", "tags": [] }, "output_type": "display_data" }, { "data": { "image/png": "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\n", "text/plain": [ "
" ] }, "metadata": { "needs_background": "light", "tags": [] }, "output_type": "display_data" }, { "data": { "image/png": "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\n", "text/plain": [ "
" ] }, "metadata": { "needs_background": "light", "tags": [] }, "output_type": "display_data" } ], "source": [ "import tensorflow as tf\n", "tf.get_logger().setLevel('ERROR') # remove annoying TF warnings\n", "\n", "conv_layers = [l for l in model.layers if type(l) == layers.Conv2D]\n", "\n", "for conv in conv_layers:\n", " conv_model = keras.models.Model(model.inputs, [conv.output])\n", " Xin = x_test[idx_1][np.newaxis]\n", " Xout1 = conv_model.predict(Xin)[0]\n", " visualize_activation(Xout1, 'image:%i - layer: %s' % (idx_1, conv.name))" ] }, { "cell_type": "code", "execution_count": 17, "metadata": { "colab": { "base_uri": "https://localhost:8080/", "height": 1000 }, "id": "Losa0sX6qlmG", "outputId": "893f42a8-a0cf-4ef6-b485-abd481a9e68f", "scrolled": false }, "outputs": [ { "data": { "image/png": "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\n", "text/plain": [ "
" ] }, "metadata": { "needs_background": "light", "tags": [] }, "output_type": "display_data" }, { "data": { "image/png": "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\n", "text/plain": [ "
" ] }, "metadata": { "needs_background": "light", "tags": [] }, "output_type": "display_data" }, { "data": { "image/png": "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\n", "text/plain": [ "
" ] }, "metadata": { "needs_background": "light", "tags": [] }, "output_type": "display_data" }, { "data": { "image/png": "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\n", "text/plain": [ "
" ] }, "metadata": { "needs_background": "light", "tags": [] }, "output_type": "display_data" }, { "data": { "image/png": "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\n", "text/plain": [ "
" ] }, "metadata": { "needs_background": "light", "tags": [] }, "output_type": "display_data" }, { "data": { "image/png": "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\n", "text/plain": [ "
" ] }, "metadata": { "needs_background": "light", "tags": [] }, "output_type": "display_data" } ], "source": [ "idx = np.random.choice(len(x_test), 1)[0]\n", "\n", "for conv in conv_layers:\n", " conv_model = keras.models.Model(model.inputs, [conv.output])\n", " Xin = x_test[idx_2][np.newaxis]\n", " Xout1 = conv_model.predict(Xin)[0]\n", " visualize_activation(Xout1, 'image:%i - layer: %s' % (idx_2, conv.name))" ] }, { "cell_type": "markdown", "metadata": { "id": "iue8W8UNqlmH" }, "source": [ "*Whereas in the first few layers, the working principle of the CNN can be somehow, in a sense, be interpreted as a simple application of image [filters from image processing](https://en.wikipedia.org/wiki/Kernel_(image_processing), this is not possible for deeper layers.\n", "To get insights into the working principle of a CNN, more sophistaced methods are needed. Check, e.g., [Exercise 12.2](Exercise_12_2.ipynb).*" ] } ], "metadata": { "accelerator": "GPU", "colab": { "name": "Exercise_12_1_solution.ipynb", "provenance": [] }, "kernelspec": { "display_name": "Python 3", "language": "python", "name": "python3" }, "language_info": { "codemirror_mode": { "name": "ipython", "version": 3 }, "file_extension": ".py", "mimetype": "text/x-python", "name": "python", "nbconvert_exporter": "python", "pygments_lexer": "ipython3", "version": "3.6.9" } }, "nbformat": 4, "nbformat_minor": 1 }